Vascular Flashcards

1
Q

3.A 30-year-old male with recurrent Transient Ischemic Attacks (TIAs) and a history of migraine with aura undergoes CT brain. Subcortical infarcts are identified raising suspicion of cerebral autosomal dominant arteriopathy with subcortical infarcts (CADASIL). Which is the most characteristically involved location for subcortical infarcts?

A. Frontal lobe

B. Centrum semiovale

C. Deep grey matter structure

D. Pons

E. Anterior temporal pole

A

E. Anterior temporal pole

A young patient with migraines, auras, TIAs or subcortical strokes should raise suspicion of CADASIL. Subcortical infarcts are characteristically in the anterior temporal pole and external capsule but may involve C, D and E.

How well did you know this?
1
Not at all
2
3
4
5
Perfectly
2
Q
  1. A 40-year-old male has a CT brain to investigate seizures. Which of the following best describes a cavernoma?

A. Most commonly occurs in the cerebellum

B. Popcorn appearance with high SI centre on T1+T2

C. Usually considerable mass effect

D. Hypodense region on CT

E. Rarely calcify

A

B. Popcorn appearance with high SI centre on T1+T2

Cavernomas are most commonly located in subcortical regions and are hypodense areas which can calcify.

They are associated with minimal mass effect/oedema.

Cavernomas can undergo haemorrhagic change & are ass with seizures.

How well did you know this?
1
Not at all
2
3
4
5
Perfectly
3
Q

@# 24. Which of the following best describes the typical appearance of intracranial haemorrhage in the late subacute phase (8-30 days)?

A. Increased T1, decreased T2

B. Isointense T1, decreased T2

C. IsointenseT1, increased T2

D. Increased T1, increased T2

E. Decreased T1, decreased T2

A

D. Increased T1, increased T2

A represents early subacute haemorrhage,

B represents acute (12-72 hours),

C represents hyperacute (less than 12 hours)

and E represents chronic haemorrhage (> 1 month).

How well did you know this?
1
Not at all
2
3
4
5
Perfectly
4
Q
  1. Considering Moyamoya disease in adults:

A. Infarct seen in cortical/subcortical areas

B. Multiple small flow voids are characteristic

C. Affects anterior circulation

D. Presentation with ischemia is more common in adults than in children

E. The supraclinoid MCA is spared

A

B. Multiple small flow voids are characteristic

Multiple flow voids are due to hypertrophied lenticulostriate arteries.

Children more often present with ischemia and infarct in cortical/subcortical areas.

Adults more often present with haemorrhage than children, but when infarcts do occur, they are most often in the deep white matter.

The disease can involve the posterior circulation.

The supraclinoid MCA is the first to be involved.

How well did you know this?
1
Not at all
2
3
4
5
Perfectly
5
Q

1) A 70-year-old woman presents with a sudden onset of right-sided hemiplegia and expressive dysphasia. She is otherwise well and has normal blood pressure on examination. Emergency CT shows a small subcortical acute haemorrhage in the inferior posterior left frontal lobe. Elsewhere, throughout the brain, there are smaller acute and subacute haemorrhages in the subcortex and a background of lacunar infarction. There is also marked brain atrophy in excess of that expected for the patient’s age. Changes consistent with diffuse leukoencephalopathy are also seen. What is the most likely underlying pathological condition?

a. malignant hypertension

b. acute disseminated encephalomyelitis

c. neurosarcoidosis

d. neuroamyloidosis

e. haemorrhagic metastases

A

d. neuroamyloidosis

Cerebral amyloid angiopathy is characterized by deposition of b-amyloid protein in cortical, subcortical and leptomeningeal vessels.

It usually occurs in sporadic form and increases in frequency and severity with increasing age.

The condition produces a wide variety of clinical symptoms and varied imaging appearances. Many cases are asymptomatic but progressive neurological symptomatology and cognitive decline may be a feature.

Chronic haemorrhage in a distinctive distribution, or catastrophic acute intracerebral/subarachnoid haemorrhage, may also occur.

Amyloid angiopathy should be strongly considered in elderly normotensive patients with spontaneous intracranial haemorrhage, particularly when associated with leukoencephalopathy and atrophy related to small-vessel cerebrovascular disease.

Definitive diagnosis is usually only made postmortem, with a presumptive diagnosis made on clinical presentation and imaging findings.

How well did you know this?
1
Not at all
2
3
4
5
Perfectly
6
Q

4) An 80-year-old man presents acutely with a dense hemiplegia. CT perfusion is performed soon after admission, which suggests that the entire involved arterial territory is beyond recovery. Which of the following options represents the most likely combination of cerebral blood flow, mean transit time and cerebral blood volume, respectively, seen within the affected brain parenchyma, compared with unaffected parenchyma?

a. increased, increased, increased

b. increased, increased, decreased

c. increased, decreased, decreased

d. decreased, decreased, decreased

e. decreased, increased, decreased

A

e. decreased, increased, decreased

Cerebral perfusion CT can distinguish viable but ischaemic tissue (the penumbra) from tissue that is beyond recovery. Other uses include evaluation of vasospasm after subarachnoid haemorrhage, assessment of cerebrovascular reserve with acetazolamide (cerebral arteriole vasodilator) in cases of vascular stenosis, evaluation of collateral flow and cerebrovascular reserve in patients having temporary balloon occlusion and assessment of microvascular permeability of intracranial neoplasms.

Cerebral perfusion CTutilizes the central volume principle. This states that CBF¼CBV/MTT, where CBF is cerebral blood flow, CBV is cerebral blood volume and MTT is mean transit time.

In practice, two CT perfusion techniques can be used. One is perfused-blood volume mapping, in which a quantity is assigned to cerebral blood volume by subtracting unenhanced CT data from CTangiographic data. It has the advantage of imaging the whole brain. The second technique is a dynamic, contrast-enhanced technique that acquires data from a limited number of axial slices, and monitors the first pass of an iodinated contrast agent bolus through the cerebral circulation. This requires an unenhanced CT brain, followed by a dynamic CT performed during injection of 50 ml of iodinated contrast (300 mg I/ml) at 4 ml/s. The first pass of contrast is observed in the brain. Cerebral perfusion is related to the concentration of iodinated contrast, which is directly related to the attenuation measured.

Several maps are produced, including the CBV, CBF and MTT.

MTT is derived from arterial and venous enhancement curves, measured by using regions of interest placed on an artery (one that is not occluded as part of an acute event) and a venous sinus.

CBV is the area under the enhancement curves,

and CBF is obtained from the central volume equation.

Differentiation of infarcted brain from penumbra is important because, while penumbra can be saved by timely thrombolysis, infarcted tissue has an increased risk of bleeding from thrombolysis with no chance of recovery.

CBF is decreased in both ischaemia and infarction, MTT is longer (.6 s) in both, while CBV is decreased in infarct but increased (or normal) in the penumbra due to cerebral autoregulatory mechanisms.

MTT is the most sensitive for stroke. So this or CBF can be used to detect stroke while CBV is used to determine whether there is infarct or reversible ischaemia.

How well did you know this?
1
Not at all
2
3
4
5
Perfectly
7
Q

7) A 77-year-old patient presents with amaurosis fugax. A carotid Doppler ultrasound scan is performed. Which of the following findings would be suggestive of an internal carotid artery stenosis of .70% (using Consensus Conference of Society of Radiologists in Ultrasound 2002 criteria)?

a. internal carotid artery peak systolic velocity of 120 cm/s

b. internal carotid artery end-diastolic velocity of 80 cm/s

c. ratio of peak systolic velocities in internal and common carotid arteries of .4.0

d. loss of flow in common carotid artery during diastole

e. no spectral broadening

A

c. ratio of peak systolic velocities in internal and common carotid arteries of .4.0

A peak systolic velocity of .230 cm/s (250 cm/s in some texts) in the internal carotid artery, an end-diastolic velocity of .100 cm/s and a ratio of peak systolic velocities between the internal and common carotid arteries of .4.0 are all features that suggest a stenosis of .70%. Loss of flow, or even reversal of flow, in the common carotid artery suggests an occlusion or very-high-grade stenosis. The grade of stenosis that is suitable for surgical rather than medical treatment has been shown to be 70%.

How well did you know this?
1
Not at all
2
3
4
5
Perfectly
8
Q

@# 8) A 62-year-old man with a history of falls and confusion undergoes MR of the brain. This demonstrates a subdural haematoma of high signal intensity on T1W images and of high signal intensity with a hypointense rim on T2W images. What is the most likely age of the haematoma?

a. ,6 hours

b. 8–72 hours

c. 3 days to 1 week

d. 1 week to several months

e. several months to several years

A

d. 1 week to several months

In the first 3–6 hours (hyperacute stage) following haemorrhage, the intact red cells contain mostly oxyhaemoglobin, which appears hyperintense on T2W images.

Desaturation occurs peripherally, forming deoxyhaemoglobin, which is seen as hypointensity on T2W images.

In the acute stage (8–72 hours), there is rapid deoxygenation of the oxyhaemoglobin to deoxyhaemoglobin, which, together with the high protein content of the clot and susceptibility effects, results in isoto hypointensity on T1W images and hypointensity on T2W images.

In the early subacute stage (37 days), oxidation of deoxyhaemoglobin to methaemoglobin occurs inside the red cell, resulting in characteristic hyperintensity on T1W images due to paramagnetic effects, and marked hypointensity on T2W images.

In the late subacute stage (1 week to months), extracellular methaemoglobin results in persistent hyperintensity on T1W images, but increasing signal intensity on T2W images, with peripheral susceptibility effects causing a low intensity rim.

In the chronic stage (months to years), iron atoms are deposited as haemosiderin and ferritin, which cause susceptibility effects resulting in low signal intensity on both T1W and T2W images.

How well did you know this?
1
Not at all
2
3
4
5
Perfectly
9
Q

9) A 75-year-old man undergoes an aortic valve replacement. His GCS remains low as the general anaesthetic effect wears off, and a new left-sided weakness is observed. Unenhanced CT of the brain is performed. The CT reveals a hypodensity (of attenuation value –30 HU) within the first segment of the right middle cerebral artery. Which of the following options is most likely to explain the patient’s abnormal neurology?

a. right middle cerebral artery thromboembolic occlusion

b. cerebral embolism as part of the fat embolism syndrome

c. right middle cerebral artery fat embolism

d. right middle cerebral artery air embolism

e. right middle cerebral artery dissection

A

c. right middle cerebral artery fat embolism

The hypodense artery sign is described, representing a single, large, macroscopic fat embolus within the middle cerebral artery, giving rise to a stroke syndrome. It may occur during cardiac surgery, resulting in dislodgement of fat from the surrounding tissue. This is distinct from the shower of microscopic fat emboli that occurs in the fat embolism syndrome.

How well did you know this?
1
Not at all
2
3
4
5
Perfectly
10
Q

24) A 60-year-old female admitted with severe, sudden-onset headache is found to have widely distributed subarachnoid haemorrhage. A saccular aneurysm is identified on CT angiography. From which of the following locations in the circle of Willis is this aneurysm most likely to arise?

a. basilar tip

b. middle cerebral artery bifurcation

c. junction of anterior cerebral and anterior communicating arteries

d. pericallosal artery

e. vertebral artery

A

c. junction of anterior cerebral and anterior communicating arteries

It is at this location that 35% of berry aneurysms are found. This is the same proportion that occurs at the junction of the internal carotid and posterior communicating arteries.

Five per cent occur at the basilar tip

and 20% at the middle cerebral artery bifurcation.

Two per cent of the population have cerebral aneurysms.

They are multiple in 20% of cases and giant in 25% (over 25 mm in diameter).

They are caused by degenerative vascular changes, trauma, infection, tumour and vasculopathy.

The incidence is increased in adult polycystic renal disease, aortic coarctation, fibromuscular dysplasia, and Marfan’s and Ehlers–Danlos syndromes.

How well did you know this?
1
Not at all
2
3
4
5
Perfectly
11
Q

33) An elderly male patient presents with signs suggesting acute middle cerebral artery infarction. Around 2 &1/2 hours after symptom onset, an unenhanced CT of the brain is performed. Among other subtle signs, the basal ganglia are obscured. Reduced perfusion through which of the following vessels best explains this sign?

a. lenticulostriate arteries

b. anterior choroidal artery

c. callosomarginal artery

d. recurrent artery of Heubner

e. angular artery

A

a. lenticulostriate arteries

The lenticulostriate arteries are vessels arising from the M1 segment of the middle cerebral artery; there are medial and lateral groups. Collectively, they supply the thalamus, caudate and lentiform nuclei.

The callosomarginal artery and the recurrent artery of Heubner are anterior cerebral artery branches. The latter provides some supply to the anterior limb of the internal capsule, and parts of the caudate nucleus and globus pallidus.

The angular artery is a cortical branch of the middle cerebral artery.

The anterior choroidal artery also supplies parts of the internal capsule and basal ganglia but is a branch of the internal carotid artery.

The nuclei of the basal ganglia are the amygdala, claustrum, lentiform and caudate nuclei, with the internal, external and extreme capsules being associated white matter tracts.

How well did you know this?
1
Not at all
2
3
4
5
Perfectly
12
Q

@# 34) An unenhanced CT scan of the brain is performed 3 hours after the onset of signs suggestive of ischaemic stroke. Which of the following image window parameters is most likely to reveal the early CT changes?

a. width 400 HU, centre 40 HU

b. width 80 HU, centre 20 HU

c. width 8 HU, centre 32 HU

d. width 0 HU, centre 0 HU

e. width 1500, centre –500 HU

A

c. width 8 HU, centre 32 HU

Loss of grey–white matter differentiation, obscuration of the lentiform nucleus and the insular ribbon sign are manifestations of cytotoxic oedema that can be seen on CTas early as 2 hours after middle cerebral artery infarction. With normal window settings (width 80 HU, centre 20 HU), the sensitivity and specificity for acute ischaemic stroke detection on unenhanced CT are 57% and 100% respectively.

The sensitivity is improved to 71% with window settings of width 8 HU and centre 32 HU.

This setting accentuates the difference between normal and oedematous brain tissue.

How well did you know this?
1
Not at all
2
3
4
5
Perfectly
13
Q

36) A patient is investigated by catheter angiogram for a giant anterior communicating artery aneurysm. Soon after the procedure, the patient experiences excessive nausea. Unenhanced CT of the brain offers no explanation, but a small bright focus is seen in the cerebellum on T2W MR images. Which of the following is the most likely explanation?

a. iodinated contrast media reaction

b. rupture of the aneurysm

c. embolism of vertebral arterial plaque

d. dissection of internal carotid artery

e. vasospasm of common carotid artery

A

c. embolism of vertebral arterial plaque

Cerebral catheter angiograms are indicated: to investigate an intracranial haemorrhage where aneurysm or arteriovenous malformation is sought; to investigate aneurysms identified on other imaging techniques; to investigate cavernous sinus syndromes, caroticocavernous fistula and venous sinus thrombosis; for preoperative assessment of tumours; and to investigate cerebral ischaemia.

Idiosyncratic and dose-related renal iodinated contrast reactions are among the complications of cerebral angiography.

Groin puncture site complications are also possible, as with any angiogram.

Complications specific to cerebral angiography include embolism of plaque, thrombus or other particulate matter. Catheter-induced spasm or dissection can also occur.

How well did you know this?
1
Not at all
2
3
4
5
Perfectly
14
Q

41) A 70-year-old man has severe rhinorrhoea and then develops a cough with haemoptysis. A chest radiograph shows a large nodule in the right lung, and a subsequent CT thorax demonstrates a cavitary, left-lung nodule. A cerebral catheter angiogram, undertaken to investigate a focal motor deficit, is most likely to reveal which of the following?

a. a giant berry aneurysm

b. multiple aneurysms with stenoses and occlusions

c. dural arteriovenous malformation

d. contrast extravasation

e. numerous collaterals supplying the anterior circle of Willis

A

b. multiple aneurysms with stenoses and occlusions

Nasal and paranasal involvement and migratory lung nodules, which can be cavitary, are typical features of Wegener’s granulomatosis. Cerebral vasculitis can also be a feature. MRI findings are non-specific and include hyperintensities on T2W images, infarcts and haemorrhage. Angiography may demonstrate occlusion, stenoses and aneurysms.

How well did you know this?
1
Not at all
2
3
4
5
Perfectly
15
Q

@# 43) A patient with multiple previous ischaemic strokes has an MRI including T2W and DWI sequences. In the left frontal lobe, there is a region of increased T2 signal. The same area is dark on the DWI and bright on the ADC map. Which of the following is the most likely age of this region of ischaemia?

a. 10 minutes

b. 30 minutes

c. 3 days

d. 2 weeks

e. 4 months

A

e. 4 months

The imaging findings are in keeping with a chronic infarct, in which the unrestricted extra water within gliosis gives a high T2 signal, dark DWI and bright ADC.

Cytotoxic oedema occurs because of early ischaemic damage to the cell membrane Na+/K+ ATPase pump. This can be seen as early as 30 minutes after symptom onset as a bright DWI region. T2W images will be normal this early and the corresponding ADC map dark.

DWI signal increases during the first week due to restricted diffusion but may remain bright for a prolonged period due to T2 ‘shine-through’. ADC will return to normal at 1–4 weeks (pseudonormalization) when restricted diffusion is matched by increasing amounts of vasogenic oedema that is not restricted.

Long-term gliosis has extra water that is not restricted. DWI can be usefully thought of as a T2 signal diminished by net water movement. Since the DWI can be bright because of T2 ‘shine-through’ rather than restricted diffusion, comparison of DWI findings with the ADC map is mandatory.

How well did you know this?
1
Not at all
2
3
4
5
Perfectly
16
Q

52) A 19-year-old man with axillary freckling has a cerebral angiogram for recurrent hemiparetic episodes. There is bilateral occlusion of the distal internal carotid arteries extending into the proximal anterior and middle cerebral arteries. This is associated with an extensive network of upper brain-stem collaterals. Which of the following is the most likely cause of the angiographic findings in this patient?

a. sickle cell disease

b. atherosclerosis

c. radiation vasculopathy

d. Moyamoya syndrome

e. amyloid angiopathy

A

d. Moyamoya syndrome

Moyamoya disease is progressive arteritis typically affecting both supraclinoid internal carotid arteries and the anterior two vessels of the circle of Willis – the anterior and middle cerebral arteries. It is termed ‘Moyamoya syndrome’ when associated with neurofibromatosis, bacterial meningitis, head trauma, tuberculosis, oral contraceptives, atherosclerosis or sickle cell anaemia. Sickle cell anaemia and atherosclerosis are not associated with axillary freckling, unlike neurofibromatosis type 1. Amyloid angiopathy should be suspected in elderly normotensive patients with multiple areas of intra-axial haemorrhage sparing the basal ganglia

How well did you know this?
1
Not at all
2
3
4
5
Perfectly
17
Q

(Ped) 52) Antenatal ultrasound scan performed at 31 weeks’ gestational age shows hydrops, for which there is no identifiable immunological cause. The scan also demonstrates cardiac enlargement and hydrocephalus. Which of the following is the most likely associated ultrasound finding?

a. unilateral megalencephaly

b. renal cysts

c. aberrant right subclavian artery

d. cerebral median tubular cystic space with high-velocity, colour Doppler flow

e. retinal tumour

A

d. cerebral median tubular cystic space with high-velocity, colour Doppler flow

The unifying diagnosis is vein of Galen aneurysm.

Three anatomical types are recognized, all of which are vascular malformations that dilate the vein of Galen, straight and transverse sinuses, and torcular herophili secondarily.

Type 1 is an arteriovenous fistula,

type 2 is an angiomatous malformation of the basal ganglia, thalami and midbrain,

and type 3 has both features.

It can be detected in utero or may present with a neonatal pattern of features (less than 1 month), an infantile pattern or an adult pattern (above 1 year).

The in utero and neonatal manifestations are due to high-output cardiac failure and mass effect of the vein of Galen aneurysm, particularly on the aqueduct. It can undergo haemorrhage and cause infarction by a steal mechanism.

How well did you know this?
1
Not at all
2
3
4
5
Perfectly
18
Q

54) A 20-year-old male is involved in an accident in which one femur is fractured. No other significant injury is revealed by CT of the brain, cervical spine, chest, abdomen and pelvis. Several hours after the injury, petechial skin haemorrhages appear, associated with respiratory distress and hypoxia. The patient complains of headache shortly before a seizure is witnessed. Repeat CT of the brain is unchanged and looks normal. Which of the following brain MRI findings best accounts for the patient’s condition?

a. diffuse low T2 signal in subarachnoid space

b. crescent-shaped, space-occupying, extra-axial, low-T2-signal abnormality

c. lenticular, space-occupying, extra-axial, low-T2-signal abnormality

d. multiple, small, non-confluent, hyperintense lesions on T2W images and DWI within the cerebral deep white and deep grey matter

e. multiple, small, non-confluent, hypointense lesions on T2Wand DWI within the cerebral deep white and deep grey matter

A

d. multiple, small, non-confluent, hyperintense lesions on T2W images and DWI within the cerebral deep white and deep grey matter

The syndrome described is the fat embolism syndrome.

This consists of the triad of acute respiratory distress with hypoxia, petechial skin haemorrhage and varying degrees of neurological dysfunction.

The last includes headache, diminished GCS, seizures and irritability.

Other features are tachycardia, fever, thrombocytopenia and anaemia. It occurs in 0.5–3.5% of long-bone fractures.

The cause is likely to be the release of bone marrow elements into the circulation as a result of trauma.

These act as emboli and they initiate a systemic inflammatory response when free fatty acids are released by the action of pulmonary lipases.

The brain lesions seen on MRI are believed to be a combination of these two insults.

Microinfarcts result from cerebral fat embolism, and oedema results from blood–brain barrier disruption which occurs because of the toxic effect of free fatty acids on brain tissue.

Acute extra-axial haemorrhage might explain the neurological features of this case but would not account for the rash or hypoxia.

How well did you know this?
1
Not at all
2
3
4
5
Perfectly
19
Q

62) Ultrasound examination of the face and neck is performed to investigate a buccal, soft-tissue mass that became noticeable during pregnancy. The lesion is heterogeneous and hypoechoic, and has sinusoidal spaces demonstrating slow flow and circular calcifications. Which of the following is the most likely diagnosis?

a. benign lymph node

b. malignant lymph node

c. pleomorphic parotid adenoma

d. arteriovenous malformation

e. venous vascular malformation

A

e. venous vascular malformation

Phleboliths if present are unique to vascular malformations.

Arterial malformations are high flow, while venous, capillary or combined malformations are low flow.

MRI is required to assess the full extent, particularly intraosseous and intracranial, of head and neck vascular malformations.

Benign lymph nodes are smooth, elliptical and hypoechoic with hilar architecture and vascularity.

Malignant lymph nodes are typically round, are hypoechoic, have no hilum and show peripheral vascularity.

Malignant lymph nodes with necrosis are seen with squamous cell and papillary cell carcinoma of thyroid.

Internal punctate Ca is seen in metastases from papillary or medullary carcinoma of thyroid.

How well did you know this?
1
Not at all
2
3
4
5
Perfectly
20
Q

68) Carotid Doppler examination is performed on a patient following a transient ischaemic attack. All other factors are in favour of surgical treatment for carotid stenosis. At what peak ICA velocity would ultrasound scan also support this management?

a. .50 cm/s

b. .75 cm/s

c. .100 cm/s

d. .200 cm/s

e. .250 cm/s

A

e. .250 cm/s

Carotid ultrasound scan can be used to assess the common, external and internal carotid arteries and the carotid bulb, including the vessel walls and the presence of plaques and stenoses. Doppler scan can display velocity profiles and allow waveform analysis and peak velocity measurement. Flow velocity increases proportionally with the degree of stenosis, except when the affected vessel is almost completely or totally occluded. Here flow velocity drops off. Stenosis above 70% is considered for surgery. This corresponds to a flow rate of greater than250 cm/s.

How well did you know this?
1
Not at all
2
3
4
5
Perfectly
21
Q

66) Following a large postpartum haemorrhage, a 25-year-old woman develops a severe headache and sudden visual field defect. What is the most likely diagnosis?

a. intracerebral haemorrhage

b. reversible posterior leukoencephalopathy

c. subarachnoid haemorrhage

d. Sheehan’s syndrome

e. vertebral artery dissection

A

d. Sheehan’s syndrome

Many of the acute neurological conditions of pregnancy occur with rising blood pressure.

Sheehan’s syndrome results from haemorrhage-induced hypotension causing pituitary infarction.

Early on, this appears as an enlarged homogeneous pituitary with low T1 signal, high T2 signal and post-contrast ring enhancement. Later, there is an empty sella.

Clinical manifestations include visual field loss, headache, ophthalmoplegia and pituitary dysfunction (diabetes insipidus).

Reversible posterior leukoencephalopathy produces cortical blindness, headaches, confusion and seizures.

Those affected are often taking immunosuppressant treatment.

Imaging features can be identical to eclampsia, peripartum cerebral angiopathy and hypertensive encephalopathy, but with a posterior predominance.

On CT, there is low attenuation change. On MRI, there is high signal on T2W/FLAIR images.

ADC maps can differentiate between likely reversible vasogenic oedema (high signal on ADC map showing unrestricted diffusion) and cytotoxic oedema (low signal due to restricted diffusion), which is more likely to progress to infarct.

Microangiopathic haemolytic anaemias, such as thrombotic thrombocytopenic purpura and haemolytic uraemic syndrome, give widespread ischaemia/infarction and haemorrhagic transformation.

There is no increased risk in pregnancy of vasculitis such as systemic lupus erythematosus, Takayasu’s syndrome or Moyamoya syndrome.

Arteriovenous malformation is no more likely to bleed in pregnancy, but there is an increased risk with arterial aneurysms.

Haemorrhage, sepsis and pulmonary embolism cause hypotension that can cause watershed infarction as well as Sheehan’s syndrome.

How well did you know this?
1
Not at all
2
3
4
5
Perfectly
22
Q

79) A hypertensive, 75-year-old female admitted with an acute stroke is found, on unenhanced CT of the brain, to have an acute basal ganglia haemorrhage. If an MRI were performed 2 weeks later, what signal characteristics would region of haemorrhage return?

a. isointense on T1W images and hyperintense on T2W images

b. hyperintense on T1W images and hyperintense on T2W images

c. isointense on T1W images and hypointense on T2W images

d. hyperintense on T1W images and hypointense on T2W images

e. hypointense on T1W images and a rim of hypointensity on T2W images

A

b. hyperintense on T1W images and hyperintense on T2W images

The MRI signal of blood depends first on whether it is moving or static, since on most sequences movement produces a signal void. When it is static, the signal returned by blood reflects the magnetic properties of the blood products and their location.

Hyperacute haemorrhage is intracellular oxyhaemoglobin that is diamagnetic, returning an isointense T1 and bright T2 signal.

At 1–2 days, deoxygenation has occurred, making the iron paramagnetic. It remains intracellular and returns an isointense T1 signal and is dark on T2W images.

At 2–7 days, haemorrhage contains paramagnetic intracellular methaemoglobin. This is bright on T1W images and dark on T2W images.

The methaemoglobin becomes extracellular from 1 week to 4 weeks, and the MRI signal is bright on both T1W and T2W sequences.

Chronic haemorrhage contains haemosiderin/ ferritin, which is ferromagnetic, is low T1 & low rim on T2. Weissleder

How well did you know this?
1
Not at all
2
3
4
5
Perfectly
23
Q

88) A 30-year-old female with a past medical history of spontaneous pulmonary embolus presents 2 weeks after giving birth with severe headache, vomiting and drowsiness. Unenhanced CT of the brain shows areas of low attenuation with sulcal effacement and small areas of parenchymal haemorrhage. These changes do not conform to an arterial distribution. What is the most likely finding on the post-contrast CT brain?

a. ring enhancement of the low-attenuation regions

b. demonstration of a basilar tip aneurysm

c. ‘empty delta’ sign

d. anterior pituitary enlargement

e. vertebral artery dissection flap

A

c. ‘empty delta’ sign

The patient has a venous sinus thrombosis causing congestion and venous infarction. Veno-occlusive disease is commoner in the first 3 weeks postpartum, especially if there is underlying hypercoagulability, including factor V Leiden abnormality, antiphospholipid antibody syndrome, and protein C, protein S or antithrombin III deficiency. On unenhanced CT, there may be hyperdense veins, grey–white matter junction haemorrhage and brain oedema. On CT venography, an ‘empty delta’ can be seen because thrombus rather than iodinated contrast occupies affected dural V. sinus.

How well did you know this?
1
Not at all
2
3
4
5
Perfectly
24
Q

91) A 31-year-old woman attends neurology clinic with a history of orthostatic headache, worst on standing, which sometimes induces vomiting, and relieved by lying down. MRI of the brain and cervical spine shows crowding of the foramen magnum due to low-lying cerebellar tonsils, elongation of the fourth ventricle, effacement of the prepontine cistern and a prominent pituitary gland. In the spine, an extradural fluid-signal collection is identified ventral to the cord. Administration of intravenous gadolinium reveals smooth areas of intracranial pachymeningeal enhancement. Which of the following diagnoses is best supported by these findings?

a. intracranial hypotension

b. intracranial hypertension

c. migraine

d. Chiari I malformation

e. Dandy–Walker malformation

A

a. intracranial hypotension

Spontaneous intracranial hypotension is a syndrome of low CSF pressure characterized by postural headaches in patients without any history of dural puncture or penetrating trauma.

It is thought to arise from an occult CSF leak due to dural defects reducing CSF volume and subsequently pressure.

Intracranial findings include downward displacement of the brain, subdural effusions, engorgement of other venous structures (including the hyperaemic pituitary) and low-lying cerebellar tonsils.

There may be flattening of the pons as the brain sags against the skull base.

Diffuse pachymeningeal enhancement is due to increased venous supply in an attempt to maintain intracranial volume and therefore pressure, according to the Monro–Kellie doctrine.

In the spine, extradural fluid collection is indicative of an occult leak.

Treatment is with an epidural blood patch where autologous blood is introduced into the extradural (epidural) space in an attempt to seal the microscopic dural defects.

How well did you know this?
1
Not at all
2
3
4
5
Perfectly
25
Q

96) A 65-year-old man has an unenhanced CT of the brain for recent onset, unilateral hand weakness. The CT shows multiple, bilateral, small supratentorial regions of low attenuation. Which of the following is the most likely associated finding?

a. unilateral carotid atherosclerosis

b. cardiac valve disease

c. coarctation of proximal descending thoracic aorta

d. subclavian stenosis

e. sinus tachycardia

A

b. cardiac valve disease

Thromboembolic cerebral infarction occurs in atrial fibrillation, cardiac valve disease, fibromuscular dysplasia, intracranial aneurysms, sickle cell disease, atherosclerosis and thrombotic thrombocytopenic purpura.

Multiple infarcts are more likely with extracranial disease and can take the form of a shower of emboli. In these cases, the distribution is typically bilateral and more commonly supratentorial.

How well did you know this?
1
Not at all
2
3
4
5
Perfectly
26
Q

12 An incidental cerebral arteriovenous malformation is noted on MR imaging. Which of the following features is associated with a better prognosis?

(a) Nidus larger than 3 cm at angiography

(b) Location in eloquent brain

(c) Superficial venous drainage

(d) Osler-Weber-Rendu syndrome

(e) Surrounding areas of low T2W signal

A

(c) Superficial venous drainage

The Spetzler-Martin scheme is used to grade AVMs. Points are allocated according to

size of the nidus (< 3 cm, 1 point; 3 - 6 cm, 2 points; > 6 cm 3 points),

location (non-eloquent brain, 0 points; eloquent brain, 1 point),

and venous drainage (superficial drainage only, 0 points; deep, 1 point).

The more points, the worse the prognosis.

Surrounding areas of low T2W signal are likely to be due to old haemorrhage.

How well did you know this?
1
Not at all
2
3
4
5
Perfectly
27
Q

16 An elderly gentleman presents with a hemiplegia. CT perfusion imaging is performed. Mean transit time (MTT), cerebral blood volume (CBV) and cerebral blood flow (CBF) were calculated. Which of following would suggest presence of an ischaemic penumbra?

(a) Increased MTT, increased CBV and increased CBF

(b) Increased MTT, decreased CBV and increased CBF

(c) Increased MTT, increased CBV and decreased CBF

(d) Decreased MTT, increased CBV and decreased CBF

(e) Decreased MTT, decreased CBV and decreased CBF

A

(c) Increased MTT, increased CBV and decreased CBF

The ischaemic penumbra is indicated by increased mean transit time (MTT) with either moderately decreased cerebral blood flow (CBF) and normal I increased cerebral blood volume (CBV) or markedly reduced CBF and moderately reduced CBV. lnfarcted tissue has an increased MTT with severe reductions in both CBF and CBV.

How well did you know this?
1
Not at all
2
3
4
5
Perfectly
28
Q

@# 18 An elderly, hypertensive man is found collapsed. CT imaging demonstrates a large intracerebral haematoma. The mean CT attenuation is 70 HU, and there are fluid/ fluid levels. How old is the haematoma likely to be?

(a) 0-2 hours

(b) 3-48 hours

(c) 3-7 days

(d) 2-4 weeks

(e) 2 months

A

(b) 3-48 hours

How well did you know this?
1
Not at all
2
3
4
5
Perfectly
29
Q

25 A 35 year old man presents with a focal weakness of his left arm. He had suffered from migraines in the past. A CT head reveals a lacunar stroke and white matter hypoattenuation, which particularly affects the sub-cortical regions a·nd extends into the temporal poles. The white matter changes did not enhance after the administration of i.v. contrast medium. Which of the following is the most likely underlying diagnosis?

(a) Amyloid angiopathy

(b) Moya moya

(c) Mitochondral cytopathy

(d) CADASIL

(e) Sickle cell disease

A

(d) CADASIL

This patient suffers from ‘Cerebral autosomal dominant arteriopathy with subcortical infarcts and leukoencephalopathy’ (CADASIL). Symptoms include migraine in the 3rd decade, recurrent stroke, and early-onset dementia.

The extension of white matter hypoattehuation into the temporal poles is virtually pathognomonic of CADASIL.

How well did you know this?
1
Not at all
2
3
4
5
Perfectly
30
Q

38 A patient presents with headache. A CT head is performed. After the administration of intravenous contrast medium, a dural pattern of enhancement is noted. Which of the following is most likely to produce this pattern?

(a) Infarction

(b) lntracranial hypotension

(c) Subarachnoid haemorrhage

(d) Sturge-Weber syndrome

(e) A normal variant.

A

(b) lntracranial hypotension

A ‘dural’, rather than ‘leptomeningeal’, pattern of enhancement can be seen in: infection, tumour, intracranial hypotension, post-operative states, idiopathic pachymeningitis, venous thrombosis, sarcoidosis, extramedullary haematopoesis, and rheumatoid arthritis.

How well did you know this?
1
Not at all
2
3
4
5
Perfectly
31
Q

40 A man is brought into A&E unconscious after near drowning. His GCS was 5/15. A CT head was performed. Which of following features would be unusual for this condition?

(a) Diffuse cerebral oedema

(b) Loss of grey-white distinction

(c) Surface blood vessels which appear dark relative to brain

(d) Relative sparing of the cerebellum

(e) Generalised decrease of the attenuation of the cerebral Parenchyma

A

(c) Surface blood vessels which appear dark relative to brain

In profound hypoxia, the surface blood vessels can appear bright relative to the brain parenchyma, an appearance which may be confused for subarachnoid blood. cerebellum is relatively spared, which leads to the ‘bright cerebellum’ sign.

How well did you know this?
1
Not at all
2
3
4
5
Perfectly
32
Q

@# 47 A pregnant lady patient presents with headache and a focal neurological deficit. A non-enhanced CT shows increased attenuation in the superior sagittal sinus. Following the administration of intravenous contrast medium, the dura surrounding the sinus enhances but the sinus itself does not. MR imaging is performed. Given the most likely diagnosis, which appearances would be unusual for this condition?

(a) Low T2W signal intensity at 2 days

(b) lso/high T1W signal intensity in 20 days

(c) High T1W signal intensity in 10 days

(d) Low T2W signal intensity in 20 days

(e) High T2W signal intensity in 10 days

A

(d) Low T2W signal intensity in 20 days

This patient has suffered a venous sinus thrombosis.

The pattern of signal intensities is as follows:

Acute (< 5 days): T1W isointense, T2W low;

Subacute: T1W high, T2W high;

Chronic: T1W isointense/ high, T2W, isointense/ high.

How well did you know this?
1
Not at all
2
3
4
5
Perfectly
33
Q

4 A 30 year old man presents with a progressive neurological deficit. A CT head demonstrates strongly enhancing, serpiginous structures. Catheter angiography subsequently visualises the lesion. What is the most likely diagnosis?

(a) Brain arteriovenous malformation

(b) Dural arteriovenous fistula

(c) Cavernous angioma

(d) Venous angioma

(e) Telangiectasia

A

(a) Brain arteriovenous malformation

Brain AVMs are the most common symptomatic vascular malformation. They typically present in 20-40-year olds with ictus from intracranial haemorrhage, progressive neurological deficit or headaches.

Imaging features include: fat CT: serpiginous, densely enhancing vessels without mass effect or oedema; at MR: a nidus of flow voids on T2W and PD sequences, at catheter angiography: grossly dilated vessels like a ‘bag of worms’.

CT imaging of a dural AVM is typically normal unless haemorrhage has occurred.

Catheter angiography of a cavernous angioma is either normal or demonstrates only a faint vascular stain.

Venous angiomata and telangiectasia are typically asymptomatic.

How well did you know this?
1
Not at all
2
3
4
5
Perfectly
34
Q

25 A 30 year old pregnant patient is found unconscious and neurological examination reveals bilateral Babinski’s sign. CT and MR imaging is suggestive of cerebral infarction. Which of the following imaging features would support an arterial rather than a venous infarct?

(a) A hyperdense superior sagittal sinus on unenhanced CT imaging

(b) Bilateral areas of increased T2W signal

(c) Prominent haemorrhage

(d) Maximal brain swelling at 4 days

(e) Intense dural enhancement

A

(d) Maximal brain swelling at 4 days

Venous occlusion progresses to infarction in approximately 50% of cases.

These infarcts do not conform to arterial territories: occlusion of the midline veins may result in bilateral infarction (low attenuation on CT and increased T2W signal on MRI).

Acute thrombus is hyperdense on pre-contrast CT imaging; however, i. v. contrast medium results in more intense enhancement of the walls of the sinuses than of their contents (the ‘delta sign’).

Haemorrhage is common. Brain swelling is prominent early in the clinical course; this is in contradistinction to arterial infarcts, where swelling is maximal at 3-5 days.

How well did you know this?
1
Not at all
2
3
4
5
Perfectly
35
Q

@# 32 An elderly gentleman has an episode of amaurosis fugax and undergoes Doppler ultrasound assessment of his carotid arteries. Which of the following features is not an indication of either internal carotid artery or common carotid artery stenosis?

(a) Peak systolic velocity of 160 cm/s

(b) Significant quantities of visible plaque

(c) End diastolic velocity of 120 cm/s

(d) Spectral broadening

(e) ICA/ CCA peak systolic velocity ratio of 1.5

A

(e) ICA/ CCA peak systolic velocity ratio of 1.5

A consensus panel has recently defined the US criteria for gauging carotid artery stenosis. Quantitative criteria include: (table)

How well did you know this?
1
Not at all
2
3
4
5
Perfectly
36
Q

33 An elderly gentleman presents with a hemiplegia. Both MR diffusion and perfusion-weighted imaging are performed. Which of the following would suggest the presence of an ischaemic penumbra?

(a) A lesion which appears the same size on diffusion and perfusion weighted imaging

(b) A lesion which appears larger on perfusion-Weighted than diffusion weighted imaging

(c) Visualisation of a lesion on perfusion-weighted imaging

(d) Change51 in the perfusion parameters over time

(e) A lesion Which appears larger on diffusion-weighted than perfusion weighted imaging

A

(b) A lesion which appears larger on perfusion-Weighted than diffusion weighted imaging

A region that shows both diffusion and perfusion abnormalities represents irreversibly infarcted tissue, while a region that shows only perfusion abnormalities and has normal diffusion represents viable but ischaemic tissue: a ‘penumbra’.

How well did you know this?
1
Not at all
2
3
4
5
Perfectly
37
Q

@# 35 An elderly, hypertensive man is found collapsed. MR imaging demonstrates a large intracerebral haematoma, with intermediate. signal intensity on T1W and high signal intensity on T2W. How old is the haematoma likely to be?

(a) 0-12 hours·· ·

(b) 12-72 hours

(c) 4-7 days

(d) 8-30 days c

(e) More than 1 month

A

(a) 0-12 hours

How well did you know this?
1
Not at all
2
3
4
5
Perfectly
38
Q

@# 38 An elderly, hypertensive man was found collapsed. MR imaging reveals a large infarct, with low signal intensity on T1W, high signal intensity on T2W, high signal on DWI and low signal on the ADC map. How old is the infarct likely to be?

(a) 0-6 hours

(b) 6 hours to 4 days

(c) 4-14 days

(d) 14-30 days

(e) More than 1 month

A

(b) 6 hours to 4 days

How well did you know this?
1
Not at all
2
3
4
5
Perfectly
39
Q

46 A 61 year old, known hypertensive man has an episode of expressive dysphasia which lasts a total of 90 minutes and resolves spontaneously in the A&E department. He is currently asymptomatic with no focal neurology. What is the most appropriate management step?

(a) Carotid imaging within 48 hours, with a view to endarterectomy within 1 week

(b) Discharge with routine GP follow-up

(c) Give 300 mg Aspirin

(d) Urgent CT (with CT angiogram/ perfusion if available)

(e) Urgent MRI, including DWI

A

(c) Give 300 mg Aspirin

The patient has had a transient ischaemic attack (TIA).

The Rothwell classification counts a score of ~4 counts as a high risk TIA; the risk ofa stroke within 4 weeks is up to 20%.

1 point is scored for each of: age (~ 60), hypertension (~140/90), diabetes, symptom duration of 10-60 minutes and speech disturbance alone; 2 points for symptoms of >60 minutes or a unilateral weakness.

The described. patient scores 5 and is therefore high risk,

however, recent NICE guidelines state the following management plan for high risk TIAs: 300 mg aspirin should be started immediately,

specialist assessment and investigation should be undertaken within 24 hours of onset of symptoms (7 days for low risk patients), and that measures for secondary prevention should be introduced.

MR imaging (with DWI) should only be performed if there is uncertainty over the diagnosis or the vascular territory involved.

The guidelines also advocate carotid imaging within 7 days with a view to endarterectomy (if appropriate) within 2 weeks.

40
Q

65 A middle aged man has acute, profound loss of a visual hemifield. MR imaging reveals an acute infarct of the left occipital pole and inferior temporal lobe. To which arterial territory does this correspond?

(a) Anterior cerebral artery

(b) Middle cerebral artery

(c) Posterior cerebral artery

(d) Posterior inferior cerebellar artery

(e) Superior cerebellar artery

A

(c) Posterior cerebral artery

The posterior cerebral artery supplies the mesial inferior temporal pole, occipital lobe, and perforators to the thalamus and midbrain.

41
Q

70 A middle aged man has an acute onset headache and subarachnoid blood is seen on a CT head. An MR angiogram is considered. Regarding cerebral MRA, which of the following is incorrect?

(a) MRA should be avoided following the endovascular ‘coiling’ of aneurysms

(b) Giant aneurysms are better visualised on CT angiography rather than 30 time of flfght MRA

(c) Aneurysms smaller than 3 mm are more difficult to detect by MRA imaging.

(d) Recent subarachnoid haemorrhage may degrade time of flight MRA imaging

(e) M RA may show aneurysms missed on a digital subtraction Angiogram

A

(a) MRA should be avoided following the endovascular ‘coiling’ of aneurysms

MRA is often used to follow ‘coiled’ aneurysms.

Giant aneurysms are rarely visualised to their full extent on 30 timeof-flight MRA because of flow in their nidus.

Time-of-flight MRA images may be degraded by the T1-effects of blood degradation products.

42
Q

9.A 36 year old woman with known polycystic kidney disease presents with a history of sudden onset headache and has signs of meningism. A CT brain reveals subarachnoid haemorrhage with haematoma within the septum pallucidum. What is the most likely site for an intracerebral aneurysm?

a. Anterior communicating artery

b. Posterior communicating artery

c. A2 segment of an anterior cerebral artery

d. Tip of the basilar artery

e. Middle cerebral artery

A

9.a. Anterior communicating artery

A clot in the septum pallucidum is virtually diagnostic of an aneurysm of the anterior communicating artery. Aneurysms of the distal anterior cerebral artery are less common.

43
Q
  1. A 68 year old male attends the emergency department after being found slumped in his chair at home by a carer. CT head shows an intraparenchymal bleed. On MRI the lesion is hyperintense on T1 and hypointense on T2-weighted imaging. Which of the following stages of haemorrhage best correlates with the MRI findings?

a. Oxyhaemaglobin

b. Deoxyhaemoglobin

c. Intracellular methaemoglobin

d. Extracellular methaemoglobin

e. Haemosiderin

A

10.c. Intracellular methaemoglobin

44
Q
  1. You are called by a paediatrician to perform a cranial ultrasound on a term neonate who requires intensive therapy following delivery. Ultrasound demonstrates a welldefined area of increased parenchymal echogenicity over the periphery of the right parietal lobe. What is the most likely diagnosis?

a. Germinal matrix haemorrhage

b. Venous infarction

c. Middle cerebral artery infarction

d. Periventricular leukomalacia

e. Subarachnoid haemorrhage

A
  1. c. Middle cerebral artery (MCA) infarction

Vascular occlusive disease is rare in the neonatal period.

When present, it is more common in the term infant and usually results from thrombosis rather than embolism.

The MCA is most commonly involved.

Aetiology includes: traumatic delivery, vasospasm due to meningitis and emboli secondary to congenital heart disease.

Ultrasound demonstrates echogenic parenchyma in the distribution of the arterial territory.

45
Q
  1. Which of the following best describes the appearance of an acute ischaemic infarct of the brain?

a. Hypointense on diffusion-weighted (DW) MRI and low apparent diffusion coefficient (ADC) values.

b. Hyperintense on DW MRI and low ADC values

c. Hypointense on DW MRI and high ADC values

d. Hyperintense on DW MRI and high ADC values

e. Isointense on DW MRI and low ADC values

A
  1. b. Hyperintense on DW MRI and low ADC values

Diffusion-weighted imaging is dependent on the motion of water molecules and provides information on tissue integrity. It is thought that interruption of cerebral blood flow results in rapid breakdown of energy metabolism and ion exchange pumps.

This causes a shift of water from the extracellular compartment into the intracellular compartment, giving cytotoxic odema.

This produces the hyperintensity on DW MR images.

ADC values tend to be low within hours of stroke and continue to decline for the next few days. They remain reduced through the first four days and then show pseudonormalisation between four and ten days.

After ten days the ADC tends to rise.

Hyper intensity on diffusion weighted (DW) MRI and low ADC values are not pathognomonic of acute infarction but sensitivities and specificities of 94% and 100% have been reported.

Other conditions including haemorrhage, abscesses, lymphoma and CreutzfeldtJacob disease have been described.

46
Q
  1. Which of the following statements best describes the features of amyloid angiopathy?

a. Solitary, peripheral haemorrhagic focus in a hypertensive elderly patient

b. Solitary, central haemorrhagic focus in a normotensive young patient

c. Multiple, peripheral haemorrhagic foci in a normotensive elderly patient

d. Multiple, central haemorrhagic foci in a normotensive elderly patient

e. Multiple, central haemorrhagic foci in a hypertensive young patient

A
  1. c. Multiple, peripheral haemorrhagic foci in a normotensive elderly patient

Multiple peripheral haemorrhages (corticomedullary junction) in normotensive elderly patients are suggestive of amyloid angiopathy.

Hypertensive haemorrhage tends to occur centrally around the basal ganglia (putamen, external capsule), thalamus and pons.

47
Q
  1. A 29 year old woman undergoes a brain CT for severe worsening headache. She is 20 weeks pregnant. No acute pathology is demonstrated. However, an incidental finding of a small right parietal arteriovenous malformation is noted. This is confirmed on subsequent MRI. You are asked to counsel the patient. What is most appropriate advice?

a. Normally the risk of haemorrhage is 2–3% per year but this increases by a factor of 10 during pregnancy. Recommend monthly follow-up with brain MRI

b. Normally the risk of haemorrhage is 10% per year but haemorrhage is likely during labour. Recommend angiography with a view to coil embolisation

c. Normally the risk of haemorrhage is 10% per year but there is no significant increase in risk during pregnancy. Recommend annual brain MRI for surveillance

d. Normally the risk of haemorrhage is 2–3% per year but there is no significant increase in risk during pregnancy. Recommend review after delivery to discuss treatment options

e. Normally the risk of haemorrhage is 15% per year but there is no significant increase in risk during pregnancy. Recommend review after delivery to discuss treatment options

A
  1. d. Normally the risk of haemorrhage is 2–3% per year but there is no significant increase in risk during pregnancy. Recommend review after delivery to discuss treatment options.

Arteriovenous malformations (AVMs) are congenital lesions consisting of multiple arteries and veins, connecting as a fistula without an intervening normal capillary bed.

The majority are solitary lesions except when associated with hereditary haemorrhagic telangiectasia.

Approximately 90% occur in the supratentorial region (parietal > frontal > temporal).

About half of AVMs present with haemorrhage (intracerebral > subarachnoid > intraventricular), approximately 25% present with focal or generalised seizures and 15% present with headache.

The overall risk of haemorrhage for an unruptured AVM is about 2–3% per year.

This increases to 6–17% in the first year following first haemorrhage and then decreases to a baseline level by the third year.

Following a second haemorrhage, the annual risk of re-bleed rises to 25% in the following year.

An approximate lifetime risk for patients with previously unruptured AVMs can be calculated as 105 minus the patient’s age in years.

Current data suggests that pregnancy does not increase the risk of an AVM haemorrhage, however when possible AVMs should be treated before pregnancy.

If the lesion is noted during pregnancy and there is no haemorrhage, treatment risks should be considered higher than the risk of haemorrhage, and treatment may be deferred until after pregnancy.

48
Q
  1. A 62 year old female presents with symptoms of amaurosis fugax. Ultrasound Doppler of her right internal carotid artery reveals a peak-systolic velocity of 2.4 m/s. What is the likely degree of stenosis?

a. 0–24%

b. 25–49%

c. 50–69%

d. >70%

e. >90%

A
  1. d. >70%

Although variation may exist between different hospitals, a suggested relationship between peak-systolic velocity (m/s) and degree of stenosis (%) is shown below.

49
Q

QUESTION 3 A 28-year-old woman suffers a 1-week episode of diarrhoea and vomiting due to the Norwalk virus. She is noted by her boyfriend to be increasingly lethargic at home and is unable to tolerate oral fluids. She then becomes confused and agitated and suffers a generalised tonic clonic seizure. Which one of the following radiological findings is most likely?

A Focal high signal on FLAIR images within the right cerebellar hemisphere

B Focal ovoid lesions of high FLAIR signal in the periventricular white

C High signal on FLAIR images in the occipital lobes

D High signal on FLAIR images in the parasagittal cortex bilaterally

E Loss of grey—white matter differentiation in the region of the basal ganglia and insula bilaterally

A

D High signal on FLAIR images in the parasagittal cortex bilaterally

This is a typical history of a patient with venous sinus thrombosis. The commonest site is the superior sagittal sinus and this can lead to venous infarcts which affect the parasagittal areas and do not conform to arterial territories.

50
Q

QUESTION 9 A 76-year-old man presents to the Emergency Department with headache and vomiting. He gives a 2-month history of transient episodes of limb weakness. His blood pressure is normal. A CT head is performed and shows several areas of frontal and parietal subcortical high attenuation. The ventricles and extra-axial CSF spaces appear normal. Which one of the following is the most likely diagnosis?

A Amyloid angiopathy

B Haemorrhagic infarcts

C Hypertensive haemorrhages

D Multifocal lymphoma

E Subarachnoid haemorrhages

A

A Amyloid angiopathy

Intracranial haemorrhage (particularly multifocal peripheral or lobar haemorrhages) in an elderly normotensive patient is suggestive of amyloid angiopathy.

51
Q

QUESTION 24 An 82-year-old woman who was previously well has had several falls in the past 2—3 months. She has become increasingly confused but there are no localising neurological signs on physical examination. Which one of the following is the most likely finding on CT?

A High attenuation in both Sylvian fissures, the basal cisterns and the lateral ventricles.

B High attenuation overlying the right frontoparietal cortex and extending into the interhemispheric space with midline shift to the left.

C Mixed attenuation areas overlying both hemispheres with normal ventricles and no midline shift.

D High attenuation lentiform collection overlying the left temporal region with effacement of the left lateral ventricle.

E Superficial areas of low density in the right frontotemporal region, containing small foci of high attenuation.

A

C Mixed attenuation areas overlying both hemispheres with normal ventricles and no midline shift.

This is a typical history of acute on chronic subdural haemorrhage.

Option A describes subarachnoid blood,

B describes an acute subdural haemorrhage,

D describes an extradural haemorrhage and

E describes cerebral contusions.

52
Q

QUESTION 33 A previously well 70-year-old man is admitted with an acute onset of left-sided weakness and dysphasia. CT shows low attenuation in the right temporoparietal region with loss of normal grey—white matter differentiation. He undergoes an MRI brain one week later. What are the most likely radiological findings?

A High FLAIR signal in the right temporoparietal region with no enhancement seen postcontrast

B High T2 signal in the right temporoparietal region with gyriform enhancement seen postcontrast

C High T2 signal in the right temporoparietal region with no enhancement seen postcontrast

D Intermediate T2 signal in the right temporoparietal region with no enhancement seen postcontrast

E Uniformly high signal on gradient echo images in the right temporoparietal Region

A

B High T2 signal in the right temporoparietal region with gyriform enhancement seen postcontrast

In the subacute phase of cerebral infarction there is disruption of the bloodbrain barrier and structural breakdown leading to oedema.

Contrast enhancement is seen on MRI in almost all cases by the end of the first week and a gyriform pattern is most characteristic.

53
Q

QUESTION 46 A 72-year-old man presented 4 days ago with right-sided weakness. CT at that time demonstrated an acute left frontoparietal haematoma. He now attends for an MRI brain. Which one of the following signal characteristics would you expect to see in the region of the haematoma?

A High signal on Tlw and T2w images

B High signal on Tlw and low signal on T2w images

C Low signal on Tlw and T2w images

D Low signal on Tlw and high signal on T2w images

E None of the above

A

B High signal on Tlw and low signal on T2w images

This describes the appearances of intracellular methaemoglobin. MR signal characteristics of intracerebral haemorrhage are predictable and are therefore a common exam topic.

54
Q

QUESTION 48 An 82-year-old man presents to the Emergency Department with a 4-day history of right homonymous hemianopia and right arm weakness. A CT head demonstrates mild low density in the left parieto-occipital cortex and he is treated conservatively for a cerebral infarct. Which of the following would be consistent with a cerebral infarct on diffusionweighted MR imaging (DWI) and apparent diffusion coefficient (ADC) map?

A High signal on ADC map and high signal on DWI after 1 week

B High signal on ADC map and low signal on DWI after 1 week

C Low signal on ADC map and high signal on DWI after 1 week

D Low signal on ADC map and high signal on DWI after 1 month

E Low signal on ADC map and low signal on DWI after 1 month

A

C Low signal on ADC map and high signal on DWI after 1 week

55
Q

QUESTION 64 A 44-year-old man is referred for a CT head to investigate headaches. This shows a hyperdense focus in the right parietal lobe containing speckled calcification. Minor patchy enhancement is seen postcontrast. An MRI brain is performed to characterise the lesion further. The lesion is of high signal on both Tlw and T2w images but has a very low signal rim. What is the most likely diagnosis?

A Capillary telangiectasia

B Cavernous angioma

C Cerebral arteriovenous malformation

D Developmental venous anomaly

E Sturge-Weber syndrome

A

B Cavernous angioma

Symptomatic haemorrhage is less common in cavernous angiomas than in dural fistulae or cerebral arteriovenous malformations and it is often an incidental finding.

56
Q

QUESTION 69 A 62-year-old man is admitted following a seizure. He gives a history of pulsatile left-sided tinnitus for 6 weeks, having undergone a left craniotomy for resection of a meningioma 4 months ago. The neurosurgeons are concerned he may have a dural arteriovenous fistula. Which one of the following investigations would be most appropriate to confirm or refute this?

A Conventional MR angiography

B Conventional MR venography

C CT angiography

D Intra-arterial catheter angiography

E Skull radiograph

A

D Intra-arterial catheter angiography

Conventional MR angiography and venography may be completely normal in the setting of dural arteriovenous fistula. Dynamic MR subtraction angiography may be helpful but intra-arterial angiography remains the gold standard.

57
Q

QUESTION 71 A 53-year-old woman who was previously well is admitted following a sudden onset of severe headache and photophobia. Her GCS at the time of admission is 13. She has a CT head which demonstrates high attenuation in the right Sylvian fissure, basal cisterns and the occipital horns of both lateral ventricles. There is also hydrocephalus. What is the most likely finding at cerebral angiography?

A Basilar tip aneurysm

B Right anterior cerebral artery aneurysm

C Right carotid artery dissection

D Right middle cerebral artery aneurysm

E Right parietal arteriovenous malformation

A

D Right middle cerebral artery aneurysm

This patient has an acute subarachnoid haemorrhage which is most commonly due to a ruptured arterial aneurysm. The distribution of blood in this case is most typical of middle cerebral artery aneurysm rupture.

58
Q
  1. A 69-year-old lady was admitted 10 days ago following an acute intracerebral haematoma diagnosed on CT. What are the most likely radiological findings on the follow-up MRI scan of brain?

A. Haematoma hypointense to grey matter on T1WI, hyperintense on T2WI.

B. Haematoma hyperintense to grey matter on both T1WI and T2WI.

C. Haematoma hyperintense to grey matter on T1WI, hypointense on T2WI.

D. Haematoma hypointense to grey matter on both T1WI and T2WI.

E. Haematoma isointense to grey matter on both T1WI and T2WI.

A
  1. B. Haematoma hyperintense to grey matter on both T1WI and T2WI.

The MRI appearances of intracranial haemorrhage are determined primarily by the state of the haemoglobin (Hb), which evolves with age. This can be staged as hyperacute (first few hours), acute (1–3 days), early subacute (3–7 days), late subacute (4–7 days to 1 month), or chronic (1 month to years). Table 6.1 illustrates the sequential signal intensity changes of the evolving intracerebral haematoma. The sequential signal intensity changes of the evolving intracerebral haematoma

59
Q

@# 6. A 52-year-old man is investigated by MRI of brain for a possible transient ischaemic attack (TIA). Focal lesions of CSF signal intensity are identified adjacent to the anterior commissures. The referring neurologist suspects that these lesions are chronic lacunar infarcts. What MRI finding suggests prominent perivascular space?

A. No restricted diffusion on DWI.

B. Hypointense on T1WI.

C. Hyperintense on T2WI.

D. Suppress on FLAIR.

E. Normal surrounding brain parenchyma.

A
  1. E. Normal surrounding brain parenchyma.

Perivascular spaces (Virchow–Robin spaces) are pial lined interstitial fluid structures that accompany penetrating arteries, but do not communicate directly with the subarachnoid space. They can occur anywhere, but typically cluster around the anterior commissure. They follow CSF signal intensity, suppress on FLAIR, and do not exhibit restricted diffusion. They can occasionally be giant when located within the midbrain.

Lacunar infarcts will also be hypointense on T1WI and hyperintense on T2WI. Restricted diffusion will be seen when acute/subacute. They are typically of increased signal on FLAIR, although will suppress if there is central encephalomalacia. A halo of surrounding high signal on T2WI and FLAIR is typical of lacunar infarction, although up to 25% of prominent perivascular spaces can also demonstrate a slight halo of increased signal. Lacunar infarcts are also more typically seen more extensive white matter disease.

60
Q

Ped) 51. A 10-year-old boy of Japanese origin presents with episodes of right transient hemiparesis and declining intellect. MRI brain is performed. Which of the following are the most likely radiological findings?

A. Multiple flow voids within the basal ganglia bilaterally.

B. Irregular beading of the left extracranial internal carotid artery.

C. Hypoplasia of the left internal carotid artery.

D. Distal left middle cerebral artery aneurysm.

E. Normal study.

A
  1. A. Multiple flow voids within the basal ganglia bilaterally.

The history and ethnic origin of the patient suggest moyamoya syndrome.

This is an idiopathic progressive arteriopathy of childhood resulting in narrowing of the distal internal carotid arteries and lenticulostriate collateralization.

This collateralization causes the multiple basal ganglia flow voids, likened to a ‘puff of smoke’ (moyamoya in Japanese) on angiography.

Secondary causes of moyamoya collateralization include neurofibromatosis type 1, post-radiation therapy and sickle cell anaemia.

Irregular beading of the extracranial internal carotid arteries is seen in fibromuscular dysplasia.

61
Q

@# 8. A 32-year-old man presents with recent onset of migraine and TIAs. He also reports some cognitive decline. Cerebral angiogram is normal. An MRI of brain reveals discrete hyperintensities in the anterior temporal poles and external capsules. What is the most likely diagnosis?

A. Cerebral autosomal dominant arteriopathy with subcortical infarcts and leukoencephalopathy (CADASIL).

B. Mitochondrial myopathy, encephalopathy, lactic acidosis, and stroke (MELAS).

C. Myoclonic epilepsy with ragged red fibres (MERRF).

D. Sporadic subcortical arteriosclerotic encephalopathy (sSAE).

E. Protein S deficiency.

A
  1. A. Cerebral autosomal dominant arteriopathy with subcortical infarcts and leukoencephalopathy (CADASIL).

This is a hereditary small vessel disease, which causes stroke in young adults.

The genetic mutation is found on chromosome 19.

Presentation can include migraine, cognitive decline, psychiatric disturbance, TIAs, and stroke, the latter usually with substantial/complete recovery after individual strokes, particularly early in the disease process.

Imaging reveals subcortical lacunar infarcts and leukoencephalopathy in young adults.

The frontal lobe has the highest lesion load, followed by the temporal lobe and insula.

Anterior temporal pole and external capsule lesions have higher sensitivity and specificity for CADASIL.

The cerebral cortex is usually spared.

MRI is the investigation of choice: CT will reveal only areas of hypodensity and angiography is normal.

sSAE is associated with hypertension and results in multiple lacunar infarcts in the lenticular nuclei, pons, internal capsule, and caudate nuclei.

MELAS and MERRF are mitochondrial disorders.

MELAS results in bilateral multiple cortical and subcortical hyperintense lesions on FLAIR images;

MERRF has a propensity for the basal ganglia and caudate nuclei, and watershed ischaemia/ infarcts are common.

Hypercoagulable states such as protein S deficiency result in cortical and lacunar infarcts of various sizes, but the cerebral angiogram is abnormal.

62
Q
  1. A 74-year-old woman is admitted with acute left-sided hemiplegia of 2 hours onset. On assessment by the stroke team she is deemed suitable for thrombolysis and referred for CT. As part of your institution’s work-up, CT perfusion (CTP) is performed following the unenhanced study. Which of the following CTP findings with regards to cerebral blood flow (CBF), cerebral blood volume (CBV) and mean transit time (MTT) are consistent with infarction?

A. Decreased CBF/decreased CBV/increased MTT.

B. Decreased CBF/increased CBV/increased MTT.

C. Decreased CBF/decreased CBV/decreased MTT.

D. Increased CBF/increased CBV/decreased MTT.

E. Increased CBF/decreased CBV/decreased MTT.

A
  1. A. Decreased CBF/decreased CBV/increased MTT.

In identifying the ischaemic penumbra, CT perfusion offers promise in improved patient selection for thrombolysis beyond a rigid time window. In the infarct core (tissue which is not salvageable) both CBF and CBV are decreased with a corresponding increase in MTT. Penumbral tissue (which is potentially recoverable by thrombolysis) exhibits a CBF/CBV mismatch with an increased CBV, but decreased CBF (and increased MTT).

An increase in CBF and CBV with decreased MTT is a feature noticed in tumours secondary to angiogenesis and microvascular permeability.

63
Q
  1. A 32-year-old woman of 38 weeks gestation presents with seizure following a headache. She is referred for CT querying venous thrombosis. Unenhanced CT brain demonstrates bilateral low attenuation change within the thalami. Given this location, where is the most likely site of thrombosis?

A. Superior sagittal sinus.

B. Transverse sinus.

C. Sigmoid sinus.

D. Straight sinus.

E. Cavernous sinus.

A
  1. D. Straight sinus.

Pregnancy is a risk factor for both venous sinus thrombosis and hypertensive haemorrhage secondary to eclampsia.

Hyperattenuating thrombus within the occluded sinus is classical on the unenhanced CT, but is seen in only 25% of cases.

On the contrast-enhanced study, a central filling defect surrounded by enhancing dura (empty delta sign) is present in over 30%.

Most of the superior cerebrum is drained by the superior sagittal sinus.

The transverse sinuses receive blood from the temporal, parietal, and occipital lobes.

The transverse sinuses drain into the sigmoid sinuses and on into the internal jugular veins.

The straight sinus forms from the confluence of the inferior sagittal sinus and vein of Galen.

The vein of Galen is part of the deep venous system, which drains the corpus callosum, basal ganglia, thalami, and upper brainstem.

Cavernous sinus thrombosis presents with proptosis and cranial nerve palsies (usually cranial nerve VI first).

The cavernous sinus receives the petrosal sinuses and middle cerebral veins.

64
Q

@# 62. A 36-year-old man is admitted following a seizure. Unenhanced CT demonstrates a right frontal mixed attenuation lesion, which avidly enhances post contrast. Multiple flow voids are seen on MRI of brain. What finding on catheter angiography differentiates arteriovenous malformation (AVM) from other vascular lesions of the brain?

A. Early venous drainage.

B. Arterial stenoses of feeder vessels.

C. External carotid transdural supply.

D. Dilated medullary veins (caput medusa).

E. Dilated cortical veins.

A
  1. A. Early venous drainage.

Brain AVMs are abnormal connections between arteries that would normally supply the brain tissue and veins that normally drain the brain, resulting in shunting with an intervening network of vessels within the brain parenchyma.

The finding of early venous drainage is important in differentiating brain AVMs from other vascular lesions.

Cortical venous drainage may be seen in superficial lesions.

Recruitment of transdural supply is observed in large lesions, although this is a more typical feature of proliferative angiopathy, which is a diffuse type of AVM.

Classically, in this condition, normal brain parenchyma is interspersed between the abnormal vessels.

Stenoses of feeder vessels are also often identified in proliferative angiopathy.

The caput medusa of dilated medullary veins is a feature of DVAs, a normal variant.

Dilated cortical veins are seen in dural arteriovenous fistulas (DAVFs), which are abnormal connections between arteries that normally supply the meninges (but not the brain) and small venules within the dura.

65
Q
  1. You are contacted by the A&E department requesting a CT brain for a 28-year-old female presenting with headache which reached peak intensity 15 minutes after onset. The headache came on during sexual activity and is associated with vomiting. Which feature in the clinical history is most predictive of subarachnoid Hge?

A. Female sex.

B. Age <40 years.

C. 15 minute onset to peak intensity.

D. Onset during sexual activity.

E. Association with vomiting.

A
  1. E. Association with vomiting.

Headache accounts for approximately 2% of all A&E department visits, yet subarachnoid haemorrhage accounts for less than 3% of these headaches. Up to half of all patients with subarachnoid haemorrhage, however, will be alert and neurologically intact. A recent study prospectively assessed neurologically intact patients presenting with headache peaking within 1 hour to determine which variables are predictive of subarachnoid haemorrhage. High-risk variables include age >40, neck pain, witnessed loss of consciousness, onset with exertion (but not sexual activity), arrival by ambulance, vomiting, and blood pressure >160/100 mmHg. The presence of one or more of these findings in a patient with an acute non-traumatic headache reaching maximum intensity within 1 hour should investigated.

66
Q
  1. A 54-year-old female is admitted following a seizure. CT of the brain demonstrates a rounded, 2-cm hyperdense lesion within the right temporal lobe, which exhibits calcification. No enhancement is seen post contrast. On follow-up MRI of brain, the lesion is of mixed intensity on T1WI. The lesion has a hypointense rim on T2WI. Prominent susceptibility effect is noted on T2* GE imaging. DWI is normal and no enhancement is demonstrated on T1WI post gadolinium. Based on the imaging findings, what is the most likely diagnosis?

A. AVM.

B. Haemorrhagic neoplasm.

C. Cavernous malformation (cavernoma).

D. Amyloid angiopathy.

E. Capillary telangiectasia.

A
  1. C. Cavernous malformation (cavernoma).

This is a benign vascular hamartoma, which can occur anywhere throughout the CNS, although spinal cord involvement is rare.

It is typically a discrete, lobulated mass <4 cm containing blood products at different stages of evolution.

On MRI they are described as having a ‘popcorn’ appearance of mixed signal intensity with a hypointense haemosiderin rim. They display little to no enhancement. Prominent susceptibility effect on T2* GE is typical.

AVMs will demonstrate multiple flow voids.

Haemorrhagic neoplasms will exhibit strong enhancement and an incomplete haemosiderin rim.

Multiple ‘black dots’ are a feature of amyloid angiopathy.

Capillary telangiectasias are poorly demarcated lesions, usually <1 cm, demonstrating ‘brush-like’ enhancement on T1WI post contrast. They are not usually identified on CT.

67
Q
  1. A 42-year-old man is admitted with sudden onset headache suspicious of subarachnoid haemorrhage. There is no loss of consciousness. Unenhanced CT demonstrates blood in the cisterns around the midbrain and into the anterior part of the interhemispheric fissure. Subsequent CT angiogram (CTA) is negative. What is advised next for this patient?

A. Lumbar puncture.

B. MRA.

C. Catheter angiography.

D. Repeat CTA in 1 week.

E. No further investigation necessary (perimesencephalic haemorrhage).

A
  1. C. Catheter angiography.

Perimesencephalic haemorrhage is confined to the cisterns around the midbrain. In some cases the only blood is found anterior to the pons. Some sedimentation into the lateral ventricles may occur, but frank haemorrhage into the ventricles and/or extension into the sylvian fissure or anterior interhemispheric fissure (as in this case) indicates arterial haemorrhage.

Catheter angiography following a negative CTA in the setting of a perimesencephalic bleed arguably does not have any extra diagnostic yield. However, catheter angiography remains the gold standard for detecting aneurysms, and in this case an anterior communicating artery aneurysm requires exclusion.

68
Q
  1. A 74-year-old man with a history of diabetes presents with a history of slurred speech and right-sided weakness, which spontaneously resolved over 1–2 hours. Initial unenhanced CT is normal. Further investigation by MRI is requested. What sequence is most sensitive for diagnosing acute ischaemia?

A. T2WI.

B. FLAIR.

C. DWI.

D. GE imaging.

E. T1WI post gadolinium

A
  1. C. DWI.

The sensitivity and specificity of DWI in detecting acute ischaemia are 88–100% and 86–100%, respectively, and DWI is established as better than other conventional MRI techniques. In addition, acute from chronic ischaemia can be differentiated. DWI is a measure of the Brownian motion of water molecules within a tissue. Two sets of images are acquired; the DWI and the quantitative ADC map.

Chronic lesions may appear bright on the DWI (T2 shine through), but only acute lesions with restricted diffusion will be dark on the ADC map.

It should be noted, however, that high signal intensity on DWI with low ADC has been reported in a variety of conditions such as abscess, lymphoma, and gliomas.

GE is useful for identifying microbleeds that may influence antithrombotic treatment.

69
Q

25 A seven-day-old infant presented with poor feeding and increased sweating. On examination he had an enlarged head, prominent forehead veins and a cranial bruit. A cranial USS revealed dilated lateral ventricles and increased vascularity posterior to the third ventricle. What is the most likely diagnosis?

a Sagittal sinus thrombosis

b Vein of Galen malformation

C Giant MCA aneurysm

d Dural fistula

e Sturge-Weber syndrome

A

25 Answer B: Vein of Galen (VoG) malformation

VoG malformation is used to describe enlarged deep venous structures of the galenic system fed by abnormal midline AV communications.

Presentation is usually early in life with high output cardiac failure and obstructive hydrocephalus. Seizures, focal neurologic deficit and haemorrhage can occur. Spontaneous thrombosis is known.

70
Q

34 A 22-year-old male presents to his GP with a history of a headache four weeks ago, which he describes as the worst headache of his life. He did not seek medical advice at the time as he was on holiday. The GP suspects he has had a subarachnoid haemorrhage. Which investigation would best determine if he did have a subarachnoid haemorrhage?

a CT cerebral angiogram

b Lumbar puncture

c Axial STIR MRI images

d Coronal FLAIR MRI images

e Axial T2*-weighted MRI images

A

34 Answer E: Axial T2 *-weighted MRI images

The history is typical of a subarachnoid haemorrhage (SAH). A month after the event the blood would have be degraded to haemosiderin, which is low signal on T2, Ti, FLAIR and T2*.

Blooming artefact occurs with T2*, which can help detect small amounts of blood and would be the most useful test to perform.

Lumbar puncture would not be helpful as it is too long since the bleed. Once a diagnosis of a subarachnoid bleed has been made then the cause for the bleed can be diagnosed using CT cranial angiogram.

71
Q

35 A 25-year-old fitness instructor presented to the Emergency Department with a two-week history of left-sided nystagmus, past pointing and vertigo. CT revealed low-density changes affecting the left superior part of the cerebellum, the left vermis and tectum. What vascular territory is involved?

a Right vertebral artery

b Left posterior inferior cerebellar artery

C Left anterior inferior cerebellar artery

d Right superior cerebellar artery

e Left superior cerebellar artery

A

35 Answer E: Left superior cerebellar artery

Left-sided cerebellar symptoms are caused by ipsilateral lesions in the cerebellum, while left-sided cerebral symptoms are due to contralateral lesions.

The most superior aspect of the cerebellar hemisphere, vermis and rectum is supplied by the superior cerebellar artery.

The posterior inferior cerebellar artery supplies the inferoposterior surface of the cerebellar hemisphere.

The anterior inferior cerebellar artery supplies the middle cerebellar peduncle, floccular region and the anterior petrosal surface of the cerebellum.

72
Q

36 A five-year-old child was admitted with diarrhoea associated with Escherichia coli infection, having stopped feeding at home. She was started on antibiotics but started complaining of a headache and then became drowsy. On CT there was high density within the right transverse sinus. The rest of the brain appeared normal and there was no hydrocephalus. What is the most likely cause for this?

a Frontal sinusitis

b Head injury

C latrogenic

d Dehydration

e Mastoiditis

A

36 Answer D: Dehydration

This child has a transverse sinus thrombosis described on the CT as high density within the dural sinus.

A contrast CT reveals a filling defect in the dural sinus, descried as the delta sign.

On MR in the acute phase there is low signal on T2 and isointense signal on T1.

Dural sinus thrombosis can result in venous infarcts which on CT appear as parenchymal haemorrhage involving both grey and white matter typically in an unusual position such as bilaterally affecting the parasagittal parenchyma.

Thirty per cent of all dural sinus thrombosis are spontaneous. The rest are either due to infection from mastoiditis, sinusitis, encephalitis and meningitis and from non-infective causes such as trauma, tumour compressing the dural sinus or from low flow states such as dehydration and congestive heart disease.

On the CT, there is no opacification of the mastoid air cells or sinuses; therefore, the most likely cause is dehydration.

73
Q

38 A 32-year-old woman had a stormy post-natal period complicated by postpartum haemorrhage (PPH). On the fifth post-partum day she developed nausea and vomiting followed by generalized tonic clonic seizures. A diagnosis of cerebral venous sinus thrombosis with likely infarction wasmade. Which of following is an unlikely feature on the MRI?

a Empty delta sign

b High signal in the cerebral venous sinuses on T2-weighted images

c Bilateral thalamic infarcts

d Basal ganglia haemorrhage

e Insular ribbon sign

A

38 Answer E: Insular ribbon sign

Thrombosis of the deep cerebral veins may cause relatively symmetric infarction of the basal ganglia, thalami and midbrain.

Involvement of the grey-white matter may occur in a non-arterial distribution.

MRI demonstrates loss of flow void in the venous sinuses and depending on the age of the clot a high signal on T2 and FLAIR images in the venous sinuses.

Pitfalls include slow flow states and arachnoid granulations.

The insular ribbon sign is seen in early arterial infarcts and indicates loss of grey-white differentiation.

74
Q

59 A 45-year-old male patient attempted suicide by taking all of his prescription tablets. Eighteen hours later he was found unconscious at home. He was stabilised and a CT head was performed the next morning. He is found to have wedge-like low densities affecting the occipito-parietal and midfrontal regions. What tablets is he most likely to have taken?

a Antiepileptics

b Antihypertensives

c Antibiotics

d Antipsychotics

e Antiflammatories

A

59 Answer B: Antihypertensives

This patient has infarcts in the regions of the watershed areas between the anterior and middle cerebral arteries and the middle and posterior cerebral arteries. This patient must have taken something to reduce the cerebral perfusion. Antihypertensive medication is the best answer.

75
Q

60 You are teaching an F1 about interpreting the difference between vasogenic and cytotoxic oedema. Which is not a feature of cytotoxic oedema?

a Sharply demarcated border

b Demonstrates restricted diffusion on DWI

C Oedema involving the cortex

d Oedema extending across the corpus callosum

e Associated with ischaemia

A

60 Answer D: Oedema extending across the corpus callosum

Vasogenic oedema is oedema associated with either a primary or secondary tumour. It is caused by increased vascular permeability. On imaging there is oedema within the white matter not affecting the cortex. It can extend across vascular territories and the corpus callosum.

Cytotoxic oedema is caused by ischaemia and results in arrested metabolism of cells. It appears as oedema conforming to a vascular territory with well-demarcated edges and involves the cortex. It does not cross the corpus callosum. There is restricted diffusion on DWI.

76
Q

(Ped) 29 A neonate is on intensive care with high-output cardiac failure. The cardiac ultrasound did not show any intracardiac or great vessel abnormalities. A cranial ultrasound is requested. What is the likely finding?

a Hydrocephalus

b Pseudotumour cerebri

C Primary neonatal brain tumour

d Vein of Galen malformation

e Intracranial haemorrhage

A

29 Answer D. Vein of Galen malformation

The vein of Galen malformation is a central arteriovenous malformation that drains directly into a secondarily enlarged vein of Galen. It is a cause of highoutput cardiac failure and should be considered if no cardiac or great vessel abnormalities are seen.

77
Q

32 A 76-year-old was scanned 14 hours post onset of left-sided weakness and facial droop. Which of the following would you not expect to find on the CT scan?

a Hyperdense MCA sign

b Loss of grey/white matter differentiation

C Obscuration of the lentiform nucleus

d Mass effect with midline shift

e Insular ribbon sign

A

32 Answer D: Mass effect with a midline shift

CT findings in stroke evolve with time. Although 60% of scans obtained within 12 hours are normal, early signs like a hyperattenuating MCA and obscured lentiform nuclei may be seen. Loss of greywhite matter interface along the lateral insula is termed the insular ribbon sign. Mass effect increases between 24 and 72 hours and is not an early sign.

78
Q

33 A 55-year-old lady was repeatedly imaged following surgical evacuation of a large right-sided extra-axial bleed sustained in a high-speed RTA. On her latest CT there was widespread cerebral oedema, with effacement of the right suprasellar cistern and medial displacement of the uncus and parahip- pocampus. Enlargement of the right CP angle cistern was also seen. What territory is she currently at most risk of infarcting?

a Right ACA territory

b Right MCA territory

C Right PCA territory

d Left MCA territory

e Left PCA territory

A

33 Answer C: Right PCA territory

Features described are those of descending transtentorial herniation where the uncus and parahippocampal gyrus protrude over the free tentorial margin.

With progressive herniation, the ipsilateral PCA may become compressed against the tentorial incisura resulting in occipital ischaemia or infarction.

Other manifestations of descending transtentorial herniation include periaqueductal necrosis, Duret haemorrhage and compressive cranial neuropathies.

79
Q

34 A 22-year-old male developed a post-coital headache and was admitted via the Emergency Department with a CT which demonstrated diffuse subarachnoid blood in the suprasellar cistern. CT cerebral angiogram and conventional cerebral catheter angiogram performed six weeks after the event were both negative. What is the most likely cause?

a Anterior communicating artery aneurysm

b Middle cerebral artery aneurysm

C Carotid dissection

d Venous bleed

e Subdural bleed

A

34 Answer D: Venous bleed

If there have been two negative angiograms, the likely cause of the cerebral bleed is venous in nature.

When there has been a single negative angiogram a repeat angiogram is usually performed after a period of time as vasospasm in an artery adjacent to an aneurysm may have occurred, leading to a false negative during the first angiogram.

80
Q

35 A 45-year-old woman was admitted with a GCS of 12. There was no history available. On CT there was bilateral parasagittal haemorrhage affecting the posterior frontal lobe and parietal lobes. The lateral ventricles were dilated and the basal cisterns partially effaced. The rest of the brain parenchyma was normal. What is the most likely diagnosis?

a Hydrocephalus

b Sagittal sinus thrombosis

C Hypertensive angiopathy

d Amyloid angiopathy

e Haemorrhage from metastatic disease

A

35 Answer B: Sagittal sinus thrombosis

Sagittal sinus thrombosis can result in venous infarcts. These typically present as bilateral parasagittal haemorrhage affecting both the grey and white matter. The patient also has hydrocephalus as a result of the venous infarct.

Hypertensive angiopathy typically produces haemorrhage within the basal ganglia.

Amyloid angiopathy results in lobar haemorrhage within the frontal and parietal lobes the deep grey matter is typically spared.

Metastasis occurs within the corticomedullary region; haemorrhage can occur within them. It would be unlikely that there were two metastases that resulted in haemorrhage.

81
Q

36 A 10-year-old child of oriental origin was being investigated for right-sided hemiplegia. MRI revealed a left MCA territory infarct. Multiple flow voids were seen within the basal ganglia and the left supraclinoid ICA appeared small in calibre on the MRA. Multiple collaterals were present in the basal cisterns and the vertebro-basilar circulation appeared normal. What do the intracranial features indicate?

a ICA dissection

b Polyarteritis nodosa

C Sarcoidosis

d Moyamoya disease

e Kawasaki’s disease

A

36 Answer D: Moyamoya disease

The Moyamoya pattern of collateral blood flow is non-specific and can be seen in slowly progressive intracranial occlusive vascular disorders such as atherosclerosis, radiation-induced angiopathy and sickle cell disease. It is classically described in the Japanese; there is occlusion of the supraclinoid ICA with progressive collateralisation to supply the ischaemic areas. Angiography demonstrates a `puff of smoke’ appearance and multiple flow voids are seen on MRI.

82
Q

37 A 66-year-old presented with symptoms of dizziness, ataxia and nausea. CT showed a low-density non-enhancing area along the right anterolateral aspect of the cerebellum. The ipsilateral cerebellar vermis and tonsil also appeared involved. A diagnosis of cerebellar infarction was made. What territory is involved?

a Superior cerebellar artery

b Posterior inferior cerebellar artery

C Anterior inferior cerebellar artery

d Basilar artery

e Right vertebral artery

A

37 Answer C: Anterior inferior cerebellar artery

The SCAs supply the entire superior surface of the cerebellar hemispheres, vermis, much of the cerebellar white matter and the dentate nuclei.

The AICAs supply the anterolateral surface of the cerebellum, the middle cerebellar peduncles, flocculus and inferolateral pons.

The PICAs supply th postero-inferior surface of the cerebellum, inferior vermis, tonsils and the posterolateral medulla.

83
Q

40 A six-month-old child was brought to the Emergency Department by his parents following a fit. The attending doctor discovered multiple bruises to his chest and pelvis and suspected nonaccidental injury. A skeletal survey revealed multiple fractures and a head CT revealed a subdural haematoma. An MR was performed and demonstrated high signal on Ti and low signal on T2. How long is it likely the injury was sustained?

a Less than 24 hours (oxyhaemoglobin)

b 1-3 days (deoxyhaemoglobin)

C 3-7 days (intracellular methaemoglobin)

d 7-14 days (extracellular methaemoglobin)

e Greater than 14 days (haemosiderin)

A

40 Answer C: 3-7 days (intracellular methaemoglobin)

The appearance of blood clot on MRI is dependent on several factors including the relaxivity and susceptibility effects of the iron-containing haemoglobin. Accurate dating of haematoma is contentious, particularly in the setting of suspected NAI, and should only be attempted by experts, although the temporal evolution of haemoglobin degradation products does occur in a predictable fashion.

84
Q

51 A 35-year-old woman is three months post partum. She had a normal vaginal delivery with an epidural as anaesthetic. Since her child’s birth she has complained of a postural headache but otherwise feels well and is not depressed. On MRI brain she was noted to have generalised thickening of the dura, which enhanced uniformly and small bilateral subdural collections. No parenchymal or bony abnormality was visible and there was no hydrocephalus. What is the most likely diagnosis?

a Dural metastasis

b En plaque meningioma

C Intracranial hypotension

d Meningitis

e Sarcoid

A

51 Answer C: Intracranial hypotension

Intracranial hypotension can occur spontaneously, post-operatively or following a lumbar puncture and presents with postural headaches.

On MRI there is dural thickening with uniform enhancement, subdural fluid collections, engorgement of the veins, sagging of the brain and increased enhancement of the pituitary gland.

The likely cause for this patient’s postural haemorrhage was an unintended dural puncture during insertion of her epidural. These are often treated by a `blood patch’ (injecting the patient’s own blood at the level of the puncture to seal the defect).

Meningitis and sarcoid can cause diffuse dural thickening with enhancement but as the patient is well this is unlikely.

En plaque meningioma and dura metastasis are unlikely as she is a bit young and the whole of the dura is affected.

85
Q

70 Regarding carotid artery Doppler, what features help differentiate the ext carotid artery from int carotid artery?

a There are no branches arising from the external carotid artery

b Usually larger than the internal carotid artery

c The external carotid artery is orientated posterolaterally towards the mastoid process

d Low resistance flow pattern is seen

e Early diastolic flow reversal

A

70 Answer E: Early diastolic flow reversal

86
Q

31 A 38-year-old male with a known left parietal AVM presented with ataxia and progressive hearing loss. A CT (pre and post contrast) did not reveal any new abnormality apart from the known AVM. On the MRI T1- and T2-weighted images showed cerebellar atrophy and FLAIR sequences revealed a hypointense rim along the cerebellar margins and the brainstem. What is the most likely diagnosis?

a Meningitis

b Chronic subarachnoid haemorrhage

C Leptomeningeal metastases

d Subdural abscess

e Meningiomatosis

A

31 Answer B: Chronic subarachnoid haemorrhage

MR imaging findings of chronic subarachnoid haemorrhage are pathognomonic. T2-weighted and gradient-echo susceptibility imaging reveal characteristic hypointensity along the pial surface/subarachnoid space of the brain and spinal cord. This finding is detected classically along the surface of the brain stem and cerebellar vermis.

Superficial siderosis of the CNS due to chronic, recurrent subarachnoid haemorrhage classically manifests with progressive bilateral sensorineural hearing loss, although ataxia and pyramidal signs are also observed. Underlying causes include vascular malformations, neoplasms and trauma.

87
Q

35 A 35-year-old woman presented with headaches and left-sided weakness. CT showed hyperdense venous sinuses. On MRI there was high signal within the superior sagittal, straight and Rt transverse venous sinuses. What further complication is she at most risk?

a Bilateral thalamic infarcts

b Bacterial abscess

c Right middle cerebral artery infarct

d Basal meningitis

e Right cerebellar infarction

A

35 Answer A: Bilateral thalamic infarcts

Imaging features suggest deep venous sinus thrombosis. Thrombosis of the deep cerebral veins may cause relatively symmetric infarction of the basal ganglia, thalami and midbrain. Involvement of the grey-white matter may occur in a non-arterial distribution. Hemorrhagic transformation may occur & brain swelling occurs earlier than in arterial infarcts.

88
Q

36 A 40-year-old man was making a good recovery following drainage of posttraumatic large-volume extra-axial bleeds. He suddenly deteriorated 24 hours following surgery and a repeat CT did not demonstrate re-accumulation of the bleeds. Bilateral tentorial hygromas were seen. MR showed inferior displacement of the midbrain and crowding of the suprasellar cistern. Following contrast there was strong enhancement of the dura. What is the most likely diagnosis?

a Meningitis

b Subarachnoid haemorrhage

C Cerebral hypotension

d Diffuse cerebral oedema

e Venous sinus thrombosis

A

36 Answer C: Cerebral hypotension

Intracranial hypotension syndrome results from inadequate CSF pressure to support the brain in the skull vault, causing the brain to sag on to the skull base.

The primary form probably arises from spontaneous arachnoid tears while the secondary form is associated with trauma, post lumbar puncture and dehydration.

MRI is the imaging of choice and shows tonsillar/insular herniation, bilateral subdural hygromas and depression of the optic chiasm.

Characteristically, diffuse dural enhancement is seen secondary to dural venous dilatation that occurs with reduced CSF volume (Monro-Kellie rule).

89
Q

37 A 42-year-old female presented with intractable facial pain. A diagnosis of trigeminal neuralgia was considered and an MRI was performed. On high resolution CISS sequences, vascular contact with the root entry zone was evident. Which vessel is most likely involved?

a Superior cerebellar artery

b Posterior inferior cerebellar artery

C Anterior inferior cerebellar artery

d Basilar artery

e Vertebral artery

A

37 Answer A: Superior cerebellar artery

The SCA is the most commonly implicated vessel in trigeminal neuralgia.

Other major arteries that maybe responsible include the AICA, basilar and vertebral arteries.

Vascular contact is best depicted on MR, which clearly delineates the cisternal course of the fifth nerve.

In the symptomatic patient, vascular contact at the root entry zone is suggestive but not necessarily diagnostic as a high incidence of vascular contact and even deformity is seen in this region in asymptomatic patients.

Treatments include microvascular decompression, radiofrequency rhizotomy and injection of the trigeminal (Gasserian) ganglion with glycerol.

90
Q

1.A 40-year-old man attends the Accident & Emergency Department with acute onset of the most severe headache of his life. CT of the head demonstrates subarachnoid haemorrhage in the left sylvian fissure and early hydrocephalus. What is the most likely site of the ruptured aneurysm?

(a) Basilar artery

(b) Right middle cerebral artery

(c) Left middle cerebral artery

(d) Anterior communicating artery

(e) Posterior cerebral artery

A
  1. (c) Left middle cerebral artery (MCA)

Rupture of a left MCA aneurysm is likely to be seen as haemorrhage in the ipsilateral sylvian fissure.

91
Q
  1. A 70-year-old male with a low score on the Glasgow coma scale is admitted to a medical assessment unit. MRI shows a large area in the left MCA distribution returning high signal on diffusion-weighted images and low signal in the corresponding area on the ADC map. The most likely diagnosis is?

(a) Acute cerebral infarct

(b) Old cerebral infarct

(c) 14-day-old cerebral infarct

(d) 7-day-old cerebral infarct

(e) Superior sagittal sinus thrombosis

A
  1. (a) Acute cerebral infarct

In the acute stage, the CT scan may be normal. On MRI, diffusion-weighted images show bright signal and low signal on the ADC map.

The signal on diffusion-weighted images increase during the first week and decreases thereafter but the signal remains hyperintense for a long time.

The ADC values decline rapidly after onset of ischaemia and subsequently increase with the ‘flip-flop’ from dark to bright 7–10 days later.

92
Q
  1. A 75-year-old woman presents with sudden onset left homonymous superior quadrantanopia. Head CT shows a subtle hypointensity in the right medial occipital lobe. On MRI, the occipital region shows high signal on FLAIR. Diffusion weighted images show high signal in the right medial occipital lobe consistent with reduced diffusion.
    What is the most likely diagnosis?

(a) Acute occipital lobe infarct in posterior cerebral artery territory

(b) Occipital lobe tumour

(c) Haemorrhage

(d) Old occipital lobe infarct

(e) Acute occipital lobe infarct in middle cerebral artery territory

A
  1. (a) Acute occipital lobe infarct in posterior cerebral artery territory

Homonymous visual field defects result from retrochiasmal lesions, which include the optic tracts, lateral geniculate body, optic radiations and visual cortex. Unilateral posterior cerebral artery infarction results in isolated hemianopia. The superior portion of the visual field projects to the inferior occipital cortex. Thus, a partial or branch occlusion of the posterior cerebral artery may lead to an isolated superior or inferior visual field defect.

93
Q
  1. A 65-year-old diabetic woman presents with dysarthria and right-sided weakness. CT shows loss of grey-white matter differentiation in the left parafalcine cortex of the frontal lobe. The arterial territory involved is?

(a) Anterior cerebral artery

(b) Middle cerebral artery

(c) Posterior cerebral artery

(d) Perforating branches of basilar artery

(e) Midbrain perforating branches

A
  1. (a) Anterior cerebral artery territory

The parafalcine cortex of the frontal lobe is supplied by the branches of anterior cerebral artery.

94
Q
  1. Which of the following statements are correct about Subarachnoid hemorrhage (SAH): (T/F)

(a) A normal CT within 24 h of onset of symptoms excludes SAH.

(b) The most common etiology is trauma.

(c) Focal interhemispheric blood suggests an anterior communicating artery aneurysm rupture.

(d) Blood in the 4th ventricles is a typical feature of posterior communicating artery aneurysm rupture.

(e) SAH detected on CT more than 1 week after the initial hemorrhage suggests recurrent bleeding.

A

Answers:
(a) Not correct
(b) Not correct
(c) Correct
(d) Not correct
(e) Correct

Explanation:
A ruptured intracranial aneurysm is the most common aetiology and accounts for 72% of cases of SAH.

A normal CT within 24 hours can exclude SAH in 90% of cases but not always.

4th ventricle blood is common with posterior inferior cerebellar artery aneurysms.

95
Q
  1. Regarding subarachnoid haemorrhage: (T/F)

(a) It occurs secondary to arteriovenous malformation in 10% of cases.

(b) It is associated with subdural haemorrhage in 20% of cases.

(c) 15-20% of patients will have multiple aneurysms.

(d) MRI is the best modality for detecting early subarachnoid haemorrhage.

(e) Cerebral vasospasm is maximal from 48 to 72 hours after the event.

A

Answers:
(a) Correct
(b) Not correct
(c) Correct
(d) Not correct
(e) Not correct

Explanation:
Subarachnoid haemorrhage is associated with subdural haemorrhage in 5% of the cases.

Initially subarachnoid haemorrhage may not be seen on MRI, hence CT is used for earlier detection of subarachnoid haemorrhage.

Cerebral vasospasm occurs at 5-17 days after the event of subarachnoid haemorrhage.